LSAT and Law School Admissions Forum

Get expert LSAT preparation and law school admissions advice from PowerScore Test Preparation.

 dshen123
  • Posts: 61
  • Joined: Nov 18, 2023
|
#113241
Will B be correct if we change it to "drugs that inhibit angiogenesis will probably prevent obesity for some obesity humans " ?
User avatar
 Dana D
PowerScore Staff
  • PowerScore Staff
  • Posts: 446
  • Joined: Feb 06, 2024
|
#113272
Hey Dshen,

Remember that we are always picking the best answer on the LSAT, so here, that's not what could be true, it's what answer choice must be true based on the facts presented to us. So no, answer choice (B) as you re-wrote it still wouldn't win out over answer choice (C), because answer choice (C) is backed by a premise in the stimulus. We know the drugs stop the creation of angiogenesis, and in rodents this prevented obestity, supporting the idea that fat tissue must need angiogenesis to grow. However, assuming this would work the same way in humans, or that the drugs would "probably" prevent obestity is just not as strongly supported.

Be very strict with yourself when answering must be true questions - you don't want to assume anything outside the facts that the stimulus presents you. Often you will have multiple answer choices that could be true, like your re-written answer choice (B), but that doesn't mean they must be true.

hope that helps!

Get the most out of your LSAT Prep Plus subscription.

Analyze and track your performance with our Testing and Analytics Package.